LSAT and Law School Admissions Forum

Get expert LSAT preparation and law school admissions advice from PowerScore Test Preparation.

 Administrator
PowerScore Staff
  • PowerScore Staff
  • Posts: 8916
  • Joined: Feb 02, 2011
|
#41650
Complete Question Explanation
(The complete setup for this game can be found here: lsat/viewtopic.php?t=6962)

The correct answer choice is (E)

If O is the only main course selected, then neither N nor P is selected. Since P and W must always be selected together, it logically follows that W cannot be selected either. With the “unassigned” group completely determined, the remaining five variables must all be selected:
PT65_D11 LG Explanations_game_#3_#14_diagram 1.png
This setup immediately validates answer choice (E) .

Answer choice (A) is incorrect, because F must be selected.

Answer choice (B) is incorrect, because G must be selected.

Answer choice (C) is incorrect, because T must be selected.

Answer choice (D) is incorrect, because V must be selected.

Answer choice (E) is the correct answer choice, because W cannot be selected.
You do not have the required permissions to view the files attached to this post.

Get the most out of your LSAT Prep Plus subscription.

Analyze and track your performance with our Testing and Analytics Package.